1.3 集合的基本运算

\({\color{Red}{欢迎到学科网下载资料学习 }}\)
【基础过关系列】2022-2023学年高一数学上学期同步知识点剖析精品讲义(人教A版2019)
\({\color{Red}{ 跟贵哥学数学,so \quad easy!}}\)

必修第一册同步拔高,难度2颗星!

基础知识

并集

概念 由所有属于集合\(A\)或属于集合\(B\)的元素所组成的集合,称为集合\(A\)\(B\)的并集.
记号 \(A∪B\)(读作:\(A\)\(B\))
符号 \(A∪B=\{x|x∈A\)\(x∈B\}\)
图形表示 image.png
性质 (1)\(A∪A=A\),即一个集合与其本身的并集是其本身;
(2)\(A∪∅=A\),即一个集合与空集的并集是其本身;
(3)\(A∪B=B∪A\),即集合的并集运算满足交换律;
(4)\(A∪B=B⟺A⊆B\),即一个集合与其子集的并集是其自身.

生活中讲的“或”,如你妈奖励你数学考试满分:今晚大餐是吃羊排或海鲜;如电视剧里女生对男朋友说:你选她或我,表达的是“选其一不可兼得”.
并列中的“或”有所不同,它指的是只要满足其中一个条件即可,比如学校搞个\(party\),要求满足\(A∪B(\)其中\(A=\{\)身高170cm以上\(\}\)\(B=\{\)长得帅\(\})\),那身高\(162cm\)的贵哥由于长得帅当然能参加了,若刘德华想参加当然也可以(满足身高\(170cm\)以上,又帅).并列中的“或”是可以两者兼得的.
 

【例】 设集合\(M=\{4,5,6,8\}\)\(N=\{3,5,7,8\}\),那么\(M∪N\)等于\(\underline{\quad \quad}\) .
解析 由并集的定义可知,\(M∪N=\{3,,4,5,6,7,8\}\).
 

【练1】 设集合\(A=\{\)菱形\(\}\)\(B=\{\)矩形\(\}\),判断正方形与\(A∪B\)的关系.
解析 正方形\(∈A∪B=\{\)菱形或矩形\(\}\)
 

【练2】 设集合\(A=\{x|-1<x≤2,x∈N\}\),集合\(B=\{2,3\}\),则\(A∪B\)等于( )
A.\(\{2\}\) \(\qquad \qquad\) B.\(\{1,2,3\}\) \(\qquad \qquad\) C.\(\{-1,0,1,2,3\}\) \(\qquad \qquad\) D.\(\{0,1,2,3\}\)
解析 \(∵A=\{x│-1<x≤2,x∈N\}=\{0,1,2\}\),集合\(B=\{2,3\}\)
\(∴A∪B=\{0,1,2,3\}\),故选:\(D\).

交集

概念 由属于集合\(A\)且属于集合\(B\)所有元素所组成的集合,称为集合\(A\)\(B\)的交集.
记号 \(A∩B(\)读作:\(A\)\(B\))
符号 \(A∩B=\{x|x∈A\)\(x∈B\}\)
图形表示 image.png
性质 (1)\(A∩A=A\)\(A∩∅=∅\)
(2)\(A∩B=B∩A\)
(3)\(A∩B⊆A\)\(A∩B⊆B\)
(4)\(A∩B=A⟺A⊆B\).

(1)交集中的“且”,是“同时满足”的意思,比如学校搞\(party\),要求满足\(A∩B(\)其中\(A=\{\)身高\(170cm\)以上\(\}\)\(B=\{\)长得帅\(\})\),那身高\(162cm\)的贵哥虽然长得帅但也遗憾出局,只有刘德华这样的人物才能参加.
(2) 当集合\(A\)和集合\(B\)无公共元素时,不能说集合\(A,B\)没有交集,而是\(A∩B=∅\)
 

【例】 设集合\(M=\{4,5,6,8\}\)\(N=\{3,5,7,8\}\),那么\(M∩N\)等于\(\underline{\quad \quad}\) .
解析 由交集的定义可知,\(M∪N=\{5,8\}\).
 

【练1】 设集合\(A=\{\)菱形\(\}\)\(B=\{\)矩形\(\}\),那么\(A∩B\)等于\(\underline{\quad \quad}\) .
解析 由交集的定义可知,\(A∩B=\{\)正方形\(\}\).
 

【练2】 设集合\(A=\{-1,1,3\}\),\(B=\{a+2,a^2+4\}\),\(A∩B=\{3\}\),则实数\(a=\)\(\underline{\quad \quad}\)
解析 因为\(A∩B=\{3\}\),根据交集的运算推理得:\(3\)是集合\(A\)和集合\(B\)的公共元素,而集合\(A\)中有\(3\),所以得到\(a+2=3\)\(a^2+4=3(\)无解,舍去\()\),解得\(a=1\)

补集

概念 对于集合\(A\),由全集\(U\)中不属于集合\(A\)的所有元素组成的集合,称为集合\(A\)相对于全集\(U\)的补集.
记号 \(C_U A(\)读作:\(A\)的补集\()\)
符号 \(C_U A=\{x|x∈U,x∉A\}\)
图形表示 image.png
性质 (1) \(C_U A⊆U\)
(2)\(C_U U=∅\)\(C_U∅=U\)
(3) \(C_U (C_U A)=A\)
(4)\(A∪(C_U A)=U\)\(A∩(C_U A)=∅\).

求集合\(A\)的补集的前提是\(A\)是全集\(U\)的子集,随着所选全集的不同,得到的补集也不同.
 

【例】 已知全集\(U=\{1,2,,3,4,5,6,7\}\)\(A=\{5,6,7\}\),则\(C_U A\)等于\(\underline{\quad \quad}\).
解析 全集\(U\)中除去集合\(A\)中元素剩下的元素是\(1,2,,3,4\),则\(C_U A=\{1,2,,3,4\}\).
 

【练】 已知全集\(U=\{1,2,3,4,5,6\}\),集合\(A=\{2,3,5\}\),集合\(B=\{1,3,4,6\}\),求集合\(A∩(C_U B)\).
解析 \(∵U=\{1,2,3,4,5,6\}\),\(B=\{1,3,4,6\}\)\(∴C_U B=\{2,5\}\)
\(∵A=\{2,3,5\}\),则\(A∩(C_U B)=\{2,5\}\)

运算律

1 交换律 \(A∪B=B∪A\)\(A∩B=B∩A\)
2 结合律 \((A∪B)∪C=A∪(B∪C)\)\((A∩B)∩C=A∩(B∩C)\)
3 分配律 \((A∩B)∪C=(A∩C)∪(B∩C)\)\((A∪B)∩C=(A∩C)∪(B∩C)\)
4 德摩根律 \(C_{U}(A \cup B)=\left(C_{U} A\right) \cap\left(C_{U} B\right)\)\(∁_U (A∩B)=(∁_UA)∪(∁_UB)\).

基本方法

【题型1】离散型集合运算

【典题1】\(A=\{x∣x^2+ax+12=0\}\)\(B=\{x∣x^2+3x+2b=0\}\)\(A∩B=\{2\}\)
(1)求\(a,b\)的值及\(A,B\)
(2)设全集\(U=A∪B\),求\((C_U A)∪(C_U B)\)
解析 (1)因为\(A∩B=\{2\}\),所以\(2∈A,2∈B\)
所以\(4+2a+12=0⇒a=-8\)\(4+6+2b=0⇒b=-5\)
所以\(A=\{x∣x^2-8x+12=0\}=\{2,6\}\)\(B=\{x∣x^2+3x-10=0\}=\{-5,2\}\)
(2)由(1)可知:\(U=A∪B=\{-5,2,6\}\)\(C_U A=\{-5\}\)\(C_U B=\{6\}\)
所以\((C_U A)∪(C_U B)=\{-5,6\}\)

巩固练习

1.设集合\(A=\{x∣x^2-2x-3=0\}\)\(B=\{x∣x^2=1\}\),则\(A∪B\)等于( )
 A.\(\{-1\}\) \(\qquad \qquad\) B.\(\{1,3\}\) \(\qquad \qquad\) C.\(\{-1,1,3\}\) \(\qquad \qquad\) D.\(\{1,-3\}\)
 

2.已知集合\(U=\{2,3,4,5,6,7\}\)\(M=\{3,4,5,7\}\)\(N=\{2,4,5,6\}\),则(  )
 A.\(M∩N=\{4,6\}\) B.\(M∪N=U\)  C.\((∁_U N)∪M=U\)_ D._\((∁_U M)∩N=N\)
 

3.已知集合\(U=\{x∈Z│-3<x<8\}\),\(∁_U M=\{-2,1,3,4,7\}\),\(N=\{-2,-1,2,4,5,7\}\),则\(M∩N\)的元素个数为(  )
 A.\(1\) \(\qquad \qquad\) B.\(2\) \(\qquad \qquad\) C.\(3\) \(\qquad \qquad\) D.\(4\)
 

4.已知集合\(A=\{2a-1,a^2,0\}\),\(B=\{1-a,a-5,9\}\),且\(A∩B=\{9\}\),则(  )
 A.\(A=\{9,25,0\}\) \(\qquad \qquad\) B.\(A=\{5,9,0\}\) \(\qquad \qquad\) C.\(A=\{-7,9,0\}\) \(\qquad \qquad\) D.\(A∪B=\{-7,9,0,25,-4\}\)
 

参考答案

  1. 答案 \(C\)
    解析 依题意,\(A=\{-1,3\},B=\{-1,1\}\),故\(A∪B=\{-1,1,3\}\),选\(C\).
  2. 答案 \(B\)
    解析 \(U=\{2,3,4,5,6,7\}\)\(M=\{3,4,5,7\}\)\(N=\{2,4,5,6\}\)
    \(M∩N=\{4,5\}\)\((∁_U N)∪M=\{3,4,5,7\}\)\((∁_U M)∩N=\{2,6\}\)\(M∪N=\{2,3,4,5,6,7\}=U\),选\(B\).
  3. 答案 \(C\)
    解析 \(U=\{-2,-1,0,1,2,3,4,5,6,7\}\),则\(M=\{-1,0,2,5,6\}\)
    \(∴M∩N=\{-1,2,5\}\)\(∴M∩N\)的元素个数为\(3\).故选:\(C\)
  4. 答案 \(C\)
    解析 \(∵A∩B=\{9\}\)\(∴9∈A\)
    \(∴2a-1=9\)\(a^2=9\)\(∴a=5\)\(a=±3\)
    \(a=3\)时,\(A=\{5,9,0\}\)\(B=\{-2,-2,9\}\),集合\(B\)错误,不满足集合元素的互异性,
    \(∴a≠3\)
    \(a=-3\)时,\(A=\{-7,9,0\}\)\(B=\{4,-8,9\}\),满足\(A∩B=\{9\}\),即\(a=-3\)成立;
    \(a=5\)时,\(A=\{9,25,0\}\)\(B=\{-4,0,9\}\)\(A∩B=\{0,9\}\)\(∴a=5\)不成立,
    综上得,\(A=\{-7,9,0\}\)\(A∪B=\{-8,-7,0,4,9\}\).故选:\(C\)

【题型2】连续型集合运算

【典题1】 已知全集\(U=R\),集合\(A=\left\{x \mid\left\{\begin{array}{l} 3-x>0 \\ 3 x+6>0 \end{array}\right\}\right.\)\(B=\{m|3>2m-1\}\)
求:(1) \(A∩B\)\(A∪B\); (2) \(C_U (A∩B)\)
解析 (1) \(\because A=\left\{x \mid\left\{\begin{array}{l} 3-x>0 \\ 3 x+6>0 \end{array}\right\}=\{x \mid-2<x<3\}\right.\)\(B=\{m|3>2m-1\}=\{m|m<2\}\)
用数轴表示集合\(A,B\),如图.

\(∴A∩B=\{x|-2<x<2\}\)\(A∪B=\{x|x<3\}\)
(2)由(1)知\(A∩B=\{x|-2<x<2\}\),如图所示.

因此\(C_U (A∩B)=\{x|x≥2,\)\(x≤-2\}\)
点拨 处理涉及不等式的集合运算,多利用数轴进行运算.
 

【典题2】集合\(A=\{x|-1<x<1\}\)\(B=\{ x|x<a\}\)
(1)若\(A∩B=∅\),求\(a\)的取值范围;
(2)若\(A∪B=\{x|x<1\}\),求\(a\)的取值范围.
解析 (1)如图所示,\(A=\{x|-1<x<1\}\)\(B=\{x|x<a\}\)\(∵A⋂B=∅\)
\(∴\)数轴上点\(a\)\(-1\)的左侧(含点\(-1\)).
\(∴a≤-1\)

(2)如图所示,\(A=\{x|-1<x<1\}\)\(B=\{x|x<a\}\)
\(∵A∪B=\{x|x<1\}\)
\(∴\)数轴上点\(a\)\(-1\)\(1\)之间(含点\(1\),但不含点\(-1\)).
\(∴-1<a≤1\)

点拨 注意对端点的处理,确定是否取得到端点.

巩固练习

1.集合\(A=\{x|-1≤x≤2\}\)\(B=\{x|x<1\}\),则\(A∩(C_R B)=\) (  )
 A.\(\{x|x>1\}\) \(\qquad \qquad\) B.\(\{x|x≥1\}\) \(\qquad \qquad\) C.\(\{x|1<x≤2\}\) \(\qquad \qquad\) D.\(\{x|1≤x≤2\}\)
 

2.已知全集\(U=R\),集合\(A=\{x|x^2-3x-4<0\}\),\(B=\{x|x-1≤0\}\),则集合\(A∩∁_U B=\) (  )
 A.\(\{x|-4<x<1\}\) \(\qquad \qquad\) B.\(\{x|-1<x≤1\}\) \(\qquad \qquad\) C.\(\{x|-1<x<4\}\) \(\qquad \qquad\) D.\(\{x|1<x<4\}\)
 

3.设全集\(U\)为实数集\(R\)\(M=\{x||x|>2\}\)\(N=\{x∣x^2-4x+3<0\}\),则图中阴影部分所表示的集合是(  )
image.png
 A.\(\{x∣x<2\}\) \(\qquad \qquad\) B.\(\{x∣-2≤x≤2\}\) \(\qquad \qquad\) C.\(\{x∣-2≤x<1\}\) \(\qquad \qquad\) D.\(\{x∣1<x≤2\}\)
 

4.已知集合\(A=\{x|x^2-x-2<0\}\),\(B=\{x|a-2<x<a\}\),若\(A∩B=\{x|-1<x<0\}\),则\(A∪B=\)(  )
 A.\((-1,2)\) \(\qquad \qquad\) B.\((0,2)\) \(\qquad \qquad\) C.\((-2,1)\) \(\qquad \qquad\) D.\((-2,2)\)
 

参考答案

  1. 答案 \(D\)
    解析 \(∵B=\{x|x<1\}\)\(∴C_R B=\{x|x≥1\}\)\(∴A∩C_R B=\{x|1≤x≤2\}\)
  2. 答案 \(D\)
    解析 \(∵A=\{x|-1<x<4\}\)\(B=\{x|x≤1\}\)\(U=R\)
    \(∴∁_U B=\{x|x>1\}\)\(∴A∩∁_U B=\{x|1<x<4\}\).故选:\(D\)
  3. 答案 \(D\)
    解析 根据图像可知阴影部分为\(N∩C_R M\)
    \(M=\{x||x∣>2\}\)可得\(C_R M=\{x∣-2≤x≤2\}\)
    \(N=\{x∣x^2-4x+3<0\}\)可得\(N=\{x∣1<x<3\}\)
    所以\(N∩C_R M=\{x∣1<x≤2\}\),故选\(D\).
  4. 答案 \(D\)
    解析 \(∵A=\{x|-1<x<2\}\)\(B=\{x|a-2<x<a\}\),且\(A∩B=\{x|-1<x<0\}\)
    \(∴a=0\)
    \(∴B=\{x|-2<x<0\}\)\(∴A∪B=(-2,2)\)
    故选:\(D\)

【题型3】综合应用

【典题1】 已知集合\(A,B\),定义\(A-B=\{x|x∈A\)\(x∉B\}\)\(A+B=\{x|x∈A\)\(x∈B\}\),则对于集合\(M,N\)下列结论一定正确的是 (  )
 A.\(M-(M-N)=N\) \(\qquad \qquad\) B.\((M-N)+(N-M)=∅\) \(\qquad \qquad\)
 C.\((M+N)-M=N\) \(\qquad \qquad\) D.\((M-N)∩(N-M)=∅\)
解析 根据题中的新定义得:\(M-N=\{x|x∈M\)\(x∉N\}\)\(N-M=\{x|x∈N\)\(x∉M\}\)
\((M-N)∩(N-M)=∅\)
故选:\(D\)
点拨 对新定义的题型,可以用具体的集合进行检验,排除一些选项,也可以用venn图理解其本质再作选择.
 

【典题2】设集合\(A=\{x|x^2=4x\}\)\(B=\{x|x^2+2(a-1)x+a^2-1=0\}\)
(1)若\(A∩B=B\),求\(a\)的取值范围; (2)若\(A∪B=B\),求\(a\)的值.
解析 (1)\(∵A=\{x|x^2=4x\}=\{0,4\}\),又\(∵A∩B=B\)\(∴B⊆A\)
①若\(B=∅\),则\(Δ=4(a-1)^2-4(a^2-1)<0\),解得\(a>1\)
因此当\(a>1\)时,\(B=∅⊆A\)
②若\(0∈B\),则\(0\)为方程\(x^2+2(a-1)x+a^2-1=0\)的一个根.
\(a^2-1=0\),解得\(a=±1\)
\(a=1\)时,\(B=\{x∣x^2=0\}=\{0\}⊆A\)
\(a=-1\)时,\(B=\{x|x^2-4x=0\}=A\)
③若\(4∈B\),则\(4\)为方程\(x^2+2(a-1)x+a^2-1=0\)的一个根,
\(a^2+8a+7=0\),解得\(a=-1\)\(a=-7\)
由②知当\(a=-1\)\(A=B\)符合题意,
\(a=-7\)时,\(B=\{x∣x^2-16x+48=0\}=\{4,12\}⊈A\)
综上可知:\(a≥1\),或\(a=-1\)
(2) \(∵A∪B=B\)\(∴A⊆B\).又\(∵A=\{0,4\}\),而\(B\)中最多有\(2\)个元素,
\(∴A=B\),即\(0,4\)为方程\(x^2+2(a-1)x+a^2-1=0\)的两个根.
\(\therefore\left\{\begin{array}{l} -2(a-1)=4 \\ a^{2}-1=0 \end{array}\right.\),解得\(a=-1\)
点拨 集合运算的性质:\(A∩B=B⟺B⊆A\)\(A∪B=B⟺A⊆B\),可用venn图理解下.
 

【典题3】设集合\(P=\{x∣x^2-x-6<0\}\),\(Q=\{x∣2a≤x≤a+3\}\)
(1)若\(P∪Q=P\),求实数\(a\)的取值范围;
(2)若\(P∩Q=ϕ\),求实数\(a\)的取值范围;
(3)若\(P∩Q=\{x∣0≤x<3\}\),求实数\(a\)的值.
解析 (1)由题意知:\(P=\{x∣-2<x<3\}\)\(∵P∪Q=P\)\(∴Q⊆P\)
①当\(Q=∅\)时,得\(2a>a+3\),解得\(a>3\)
②当\(Q≠∅\)时,得\(-2<2a≤a+3<3\),解得\(-1<a<0\)
综上,\(a∈(-1,0)∪(3,+∞)\)
(2)①当\(Q=∅\)时,得\(2a>a+3\),解得\(a>3\)
②当\(Q≠∅\)时,得\(\left\{\begin{array}{l} 2 a \leq a+3, \\ a+3 \leq-2 \text { 或 } 2 a \geq 3 \end{array}\right.\),解得\(a≤-5\)\(\dfrac{3}{2} \leq a \leq 3\)
综上,\(a \in(-\infty,-5] \cup\left[\dfrac{3}{2},+\infty\right)\)
\(P∩Q=\{x∣0≤x<3\}\),则\(a=0\)
点拨 注意利用数轴配合分析,留心端点的取舍.
 

【典题4】 已知集合\(A=\{x|2m-1<x<3m+2\}\)\(B=\{x|x≤-2,\)\(x≥5\}\),是否存在实数\(m\),使\(A∩B≠∅\)?若存在,求实数\(m\)的取值范围;若不存在,请说明理由.
解析 \(A∩B=∅\),分\(A=∅\)\(A≠∅\)讨论:
(1)若\(A=∅\),则\(2m-1≥3m+2\),解得\(m≤-3\),此时\(A∩B=∅\)
(2)若\(A≠∅\),要使\(A∩B=∅\),则应有
\(\left\{\begin{array}{l} 2 m-1<3 m+2 \\ 2 m-1 \geq-2 \\ 3 m+2 \leq 5 \end{array}\right.\)\(\left\{\begin{array}{l} m>-3 \\ m \geq-\dfrac{1}{2} \\ m \leq 1 \end{array}\right.\),所以\(-\dfrac{1}{2} \leq m \leq 1\)
综上,当\(A∩B=∅\)时,\(m≤-3\)\(-\dfrac{1}{2} \leq m \leq 1\)
\(m>1\)\(-3<m<-\dfrac{1}{2}\)时,\(A∩B≠∅\)
点拨 补集思想,\(A∩B≠∅\)难以处理,可先求\(A∩B=∅\)时对应\(m\)的范围,其补集即是所求.

巩固练习

1.某班举行数理化竞赛,每人至少参加一科,已知参加数学竞赛的有\(27\)人,参加物理竞赛的有\(25\)人,参加化学竞赛的有\(27\)人,其中参加数学、物理两科的有\(10\)人,参加物理、化学两科的有\(7\)人,参加数学、化学两科的有\(11\)人,而参加数、理、化三科的有\(4\)人,则全班人数是\(\underline{\quad \quad}\)
 

2.设\(A,B\)\(R\)中两个子集,对于\(x∈R\),定义:\(m=\left\{\begin{array}{l} 0, x \notin A, \\ 1, x \in A, \end{array} n=\left\{\begin{array}{l} 0, x \notin B \\ 1, x \in B \end{array}\right.\right.\)
①若\(A⊆B\).则对任意\(x∈R\),\(m(1-n)=\) \(\underline{\quad \quad}\)
②若对任意\(x∈R\),\(m+n=1\),则\(A,B\)的关系为\(\underline{\quad \quad}\)
 

3.已知\(A=\{x|x^2+ax+b=0\}\)\(B=\{x|x^2+cx+15=0\}\)\(A∪B=\{3,5\}\)\(A∩B=\{3\}\),则实数\(a=\)\(\underline{\quad \quad}\)\(b=\)\(\underline{\quad \quad}\)\(c=\)\(\underline{\quad \quad}\)
 

4.设\(A=\{x|x^2+4x=0\}\)\(B=\{x|x^2+2(a+1)x+a^2-1=0\}\),其中\(x∈R\),如果\(A∩B=B\),则实数\(a\)的取值范围\(\underline{\quad \quad}\)
 

5.已知集合\(A=\{x|-2<x-1<2\}\),集合\(B=\{x|x^2-(2a-1)x+a^2-a=0\}\)
(1)若\(A∩B=\{2\}\),求\(a\)的值;
(2)若\(A∪B=A\),求\(a\)的取值范围.
 
 

6.对于正整数集合\(A=\{a_1,a_2,…,a_n\}(n∈N^*,n≥3)\),如果去掉其中任意一个元素\(a_i (i=1,2,…,n)\)之后,剩余的所有元素组成的集合都能分为两个交集为空集的集合,且这两个集合的所有元素之和相等,就称集合\(A\)为“和谐集”.
(1)判断集合\(\{1,2,3,4,5\}\)是否为“和谐集”,并说明理由;
(2)求证:集合\(\{1,3,5,7,9,11,13\}\)是“和谐集”;
(3)求证:若集合\(A\)是“和谐集”,则集合\(A\)中元素个数为奇数.
 
 

参考答案

  1. 答案 \(55\)
    解析 设参加数学、物理、化学三科竞赛的同学组成的集合分别为\(A,B,C\),由题意可知集合\(A,B,C\)中的元素个数分别为\(27,25,27\),集合\(A∩B\)\(B∩C\)\(A∩C\)\(A∩B∩C\)中的元素个数分别为\(10,7,11,4\).画出\(Venn\)图如图所示,

    由图可知全班人数为\(10+13+12+6+4+7+3=55\)(人).

  2. 答案 \(0\)\(A=∁_R B\)
    解析 \(∵A⊆B\).则\(x∉A\)时,\(m=0\)\(m(1-n)=0\)
    \(x∈A\)时,必有\(x∈B\)\(∴m=n=1\)\(m(1-n)=0\)
    综上可得:\(m(1-n)=0\)
    ②对任意\(x∈R\)\(m+n=1\),则\(m,n\)的值一个为\(0\),另一个为\(1\),即\(x∈A\)时,必有\(x∉B\),或\(x∈B\)时,必有\(x∉A\)
    \(∴A,B\)的关系为\(A=∁_R B\)
    故答案为:\(0\)\(A=∁_R B\)

  3. 答案 \(a=-6,b=9,c=-8\).
    解析 \(∵A∩B=\{3\}\)
    \(∴\)\(9+3c+15=0\),解得\(c=-8\).
    \(x^2-8x+15=0\),解得\(B=\{3,5\}\),故\(A=\{3\}\)
    \(a^2-4b=0\),解得\(a=-6\)\(b=9\).
    综上知,\(a=-6,b=9,c=-8\).

  4. 答案 \((-∞,-1]∪\{1\}\)
    解析 \(A\)中方程变形得:\(x(x+4)=0\)
    解得:\(x=0\)\(x=-4\),即\(A=\{-4,0\}\)
    \(B=\{x|x^2+2(a+1)x+a^2-1=0\}\),其中\(x∈R\),且\(A∩B=B\)
    分两种情况考虑:
    \(B=∅\)时,\(Δ=4(a+1)^2-4(a^2-1)=8a+8<0\),即\(a≤-1\),满足题意;
    \(B≠∅\)\(Δ=4(a+1)^2-4(a^2-1)=8a+8≥0\),即\(a≥-1\)
    此时把\(x=-4\)代入得:\(16-8a-8+a^2-1=0\),即\(a=-1\)\(a=-7\)(舍去);
    \(x=0\)代入得:\(a=1\)\(-1\)
    综上,\(a\)的范围为\((-∞,-1]∪\{1\}\)

  5. 答案 (1) \(a=2\)\(a=3\) (2) \((0,3)\)
    解析 (1)\(A=\{x|-1<x<3\}\)\(B=\{a,a-1\}\)
    \(∵A∩B=\{2\}\)\(∴2∈B\)
    \(∴a=2\)\(a-1=2\),即\(a=2\)\(a=3\)
    \(a=2\)时,\(B=\{1,2\}\)\(∴A∩B=\{1,2\}\),不满足\(A∩B=\{2\}\)\(a=2\)舍去,
    \(∴a=3\)
    (2)\(∵A∪B=A\)\(∴B⊆A\)
    \(\therefore\left\{\begin{array}{l} -1<a<3 \\ -1<a-1<3 \end{array}\right.\),解得\(0<a<3\)
    \(∴a\)的取值范围为\((0,3)\)

  6. 答案 (1) \(\{1,2,3,4,5\}\)不是“和谐集” (2) 略 (3)略
    解析 (1)对于集合\(\{1,2,3,4,5\}\)
    当去掉元素\(2\)时,剩余的所有元素之和为\(13\)
    不能分为两个交集为空集且这两个集合的所有元素之和相等的集合,
    所以集合\(\{1,2,3,4,5\}\)不是“和谐集”.
    (2)证明:设\(A=\{1,3,5,7,9,11,13\}\)
    当去掉元素\(1\)时,有\(3+5+7+9=11+13\)
    当去掉元素\(3\)时,有\(1+9+13=5+7+11\)
    当去掉元素\(5\)时,有\(9+13=1+3+7+11\)
    当去掉元素\(7\)时,有\(1+9+11=3+5+13\)
    当去掉元素\(9\)时,有\(1+3+5+11=7+13\)
    当去掉元素\(11\)时,有\(3+7+9=1+5+13\)
    当去掉元素\(13\)时,有\(1+3+5+9=7+11\)
    所以集合\(A=\{1,3,5,7,9,11,13\}\)是“和谐集”.
    (3)证明:设“和谐集”\(A=\left\{a_{1}, \quad a_{2}, \ldots, \quad a_{n}\right\}\)所有元素之和为\(M\)
    由题可知,\(M-a_i (i=1,2,…,n)\)均为偶数,
    因此\(a_i (i=1,2,…,n)\)的奇偶性相同.
    \((ⅰ)\)如果\(M\)为奇数,则\(a_i (i=1,2,…,n)\)也均为奇数,
    由于\(M=a_1+a_2+⋯+a_n\),所以\(n\)为奇数.
    \((ⅱ)\)如果\(M\)为偶数,则\(a_i (i=1,2,…,n)\)均为偶数,
    此时设\(a_i=2b_i\),则\(\{b_1,b_2,…,b_n \}\)也是“和谐集”.
    重复上述操作有限次,便可得各项均为奇数的“和谐集”.
    此时各项之和也为奇数,集合\(A\)中元素个数为奇数.
    综上所述,集合\(A\)中元素个数为奇数.

分层练习

【A组---基础题】

1.设集合\(A=\{4,5,7,9\}\)\(B=\{3,4,7,8,9\}\),全集\(U=A∪B\),则集合\(∁_U (A∩B)\)中的元素共有(  )
 A.\(3\)\(\qquad \qquad\) B.\(4\)个 \(\qquad \qquad\) C.\(5\)\(\qquad \qquad\) D.\(6\)
 

2.集合\(A=\{0,2,a\}\)\(B=\{1,a^2 \}\),若\(A∪B=\{0,1,2,4,16\}\),则\(a\)的值为( )
 A.\(0\) \(\qquad \qquad\) B.\(1\) \(\qquad \qquad\) C.\(2\) \(\qquad \qquad\) D.\(4\)
 

3.已知全集\(U=R\),集合\(A=\{0,1,2,3,4\}\),\(B=\{x∣x>2\)\(x<0\}\),则图中阴影部分表示的集合为( )
image.png
 A.\(\{0,1,2\}\) \(\qquad \qquad\) B.\(\{1,2\}\) \(\qquad \qquad\) C.\(\{3,4\}\) \(\qquad \qquad\) D.\(\{0,3,4\}\)
 

4.设集合\(A=\{x|-1≤x≤2\}\)\(B=\{x|0≤x≤4\}\),则\(A∩B\)等于(  )
 A.\(\{x|0≤x≤2\}\) \(\qquad \qquad\) B.\(\{x|1≤x≤2\}\) \(\qquad \qquad\)
 C.\(\{x|0≤x≤4\}\) \(\qquad \qquad\) D.\(\{x|1≤x≤4\}\)

5.设集合\(A=\{-4,2m-1,m^2 \}\)\(B=\{9,m-5,1-m\}\),又\(A∩B=\{9\}\),求实数\(m=\)\(\underline{\quad \quad}\)
 

6.已知集合\(A=\{x|x^2-x-2<0\}\),\(B=\{x|a-2<x<a\}\),若\(A∩B=\{x|-1<x<0\}\),则\(A∪B=\)\(\underline{\quad \quad}\) .
 

7.已知集合\(A=\{x|x^2-(2+a)x+2a=0\}\)\(B=\{2,5,a^2+5a-12\}\)
(1)若\(3∈A\),求实数\(a\)的值;
(2)若\(∁_B A=\{5\}\),求实数\(a\)的值.
 
 

8.已知集合\(A=\{x∣x^2+x-6≥0\}\)\(B=\{x∣x^2-6x+5<0\}\)\(C=\{x∣m-1≤x≤2m\}\)
(1)求\(A∩B\)\((C_R A)∪B\)
(2)若\(B∩C=C\),求实数\(m\)的取值范围.
 
 

9.已知集合\(A=\{x|-2<x-1<2\}\),集合\(B=\{x|x^2-(2a-1)x+a^2-a=0\}\)
(1)若\(A∩B=\{2\}\),求\(a\)的值;
(2)若\(A∪B=A\),求\(a\)的取值范围.
 
 

参考答案

  1. 答案 \(A\)
    解析 \(U=A∪B=\{3,4,5,7,8,9\}\)\(A∩B=\{4,7,9\}\)\(∴∁_U (A∩B)=\{3,5,8\}\)

  2. 答案 \(D\)
    解析 \(∵A=\{0,2,a\}\)\(B=\{1,a^2 \}\)\(A∪B=\{0,1,2,4,16\}\)
    \(\therefore\left\{\begin{array}{l} a^{2}=16 \\ a=4 \end{array}\right.\),\(∴a=4\).

  3. 答案 A
    解析 \(∵\)全集\(U=R\),集合\(A=\{0,1,2,3,4\}\),\(B=\{x∣x>2\)\(x<0\}\)
    \(∴C_U B=\{x∣0≤x≤2\}\)\(∴\)图中阴影部分表示的集合为\(A∩C_U B=\{0,1,2\}\),故选\(A\)

  4. 答案 \(A\)
    解析 在数轴上表示出集合\(A\)\(B\),如下图.

    则由交集的定义可得\(A∩B=\{x∣0≤x≤2\}\). 答案 :\(A\)

  5. 答案 \(-3\)
    解析 \(∵A∩B=\{9\}\)\(∴9∈A\)\(9∈B\)
    \(2m-1=9\),即\(m=5\)代入得\(A=\{-4,9,25\}\),\(B=\{9,0,-4\}\)
    \(∴A∩B=\{-4,9\}\)矛盾.
    \(m^2=9\),即\(m=±3\)
    \(m=3\)时,\(A=\{-4,5,9\}\)\(B=\{9,-2,-2\}\)矛盾(集合\(B\)中元素不互异).
    \(m=-3\)时,\(A=\{-4,-7,9\}\)\(B=\{9,-8,4\}\),有\(A∩B=\{9\}\)适合,
    由上述知:\(m=-3\)

  6. 答案 \((-2,2)\)
    解析 \(∵A=\{x|-1<x<2\}\)\(B=\{x|a-2<x<a\}\),且\(A∩B=\{x|-1<x<0\}\)
    \(∴a=0\)
    \(∴B=\{x|-2<x<0\}\)
    \(∴A∪B=(-2,2)\)

  7. 答案 (1) \(3\) (2) \(-6\)
    解析 (1)因为\(3∈A\)\(A=\{x|(x-2)(x-a)=0\}\),所以\(a=3\)
    (2)因为\(∁_B A=\{5\}\),所以\(A\)中有两个元素,即\(A=\{2,a\}\),所以\(a^2+5a-12=a\)
    解得\(a=2\)\(a=-6\),由元素的互异性可得,\(a=-6\)

  8. ** 答案 ** (1)\(\{x∣2≤x<5\}\), \(\{x∣-3<x<5\}\) (2) \((-\infty,-1) \cup\left(2, \dfrac{5}{2}\right)\)
    解析 (1)\(∵A=\{x∣x≤-3\)\(x≥2\}\)\(B=\{x∣1<x<5\}\)
    \(∴A∩B=\{x∣2≤x<5\}\)\(C_R A=\{x∣-3<x<2\}\)
    \(∴(C_R A)∪B=\{x∣-3<x<5\}\)
    (2)\(∵B∩C=C\)\(∴C⊆B\)
    ①当\(C=∅\)时,\(m-1>2m\),即\(m<-1\)
    ②当\(C≠∅\)时,\(\left\{\begin{array}{c} m-1 \leq 2 m \\ m-1>1 \\ 2 m<5 \end{array}\right.\)\(\therefore 2<m<\dfrac{5}{2}\)
    综上所述:\(m\)的取值范围是\((-\infty,-1) \cup\left(2, \dfrac{5}{2}\right)\)

  9. 答案 (1)\(3\) (2) \((0,3)\)
    解析 (1)\(A=\{x|-1<x<3\}\)\(B=\{a,a-1\}\)
    \(∵A∩B=\{2\}\)\(∴2∈B\)
    \(∴a=2\)\(a-1=2\),即\(a=2\)\(a=3\)
    \(a=2\)时,\(B=\{1,2\}\)\(∴A∩B=\{1,2\}\),不满足\(A∩B=\{2\}\)\(a=2\)舍去,
    \(∴a=3\)
    (2)\(∵A∪B=A\)\(∴B⊆A\)
    \(\therefore\left\{\begin{array}{l} -1<a<3 \\ -1<a-1<3 \end{array}\right.\),解得\(0<a<3\)
    \(∴a\)的取值范围为\((0,3)\)

【B组---提高题】

1.定义集合的商集运算为\(\dfrac{A}{B}=\left\{x \mid x=\dfrac{m}{n}, m \in A, n \in B\right\}\),已知集合\(S=\{2,4,6\}\)\(T=\{x|x=\dfrac{k}{2}-1,k∈S\}\),则集合\(\dfrac{S}{T} \cup T\)中的元素个数为(  )
 A.\(5\) \(\qquad \qquad\) B.\(6\) \(\qquad \qquad\) C.\(7\) \(\qquad \qquad\) D.\(8\)
 

2.已知集合\(A\)中有\(10\)个元素,\(B\)中有\(6\)个元素,全集\(U\)\(18\)个元素,\(A∩B≠∅\).设集合\((∁_U A)∩(∁_U B)\)\(x\)个元素,则\(x\)的取值范围是(  )
 A.\(3≤x≤8\),且\(x∈N\) \(\qquad \qquad\) B.\(2≤x≤8\),且\(x∈N\)
 C.\(8≤x≤12\),且\(x∈N\) \(\qquad \qquad\) D.\(10≤x≤15\),且\(x∈N\)
 

3.设集合\(A=\{x|x^2-(a+3)x+3a=0\}\)\(B=\{x|x^2-5x+4=0\}\),集合\(A∪B\)中所有元素之和为\(8\),则实数\(a\)的取值集合为\(\underline{\quad \quad}\)
 

4.设\(M=\left\{x \mid m \leq x \leq m+\dfrac{1}{3}\right\}\),\(N=\left\{x \mid n-\dfrac{3}{4} \leq x \leq n\right\}\)都是\(\{x|0≤x≤1\}\)的子集,如果\(b-a\)叫做集合\(\{x|a≤x≤b\}\)的长度,则集合\(M∩N\)的长度的最小值是\(\underline{\quad \quad}\).
 

5.设\(A=\{x|x^2+4x≤0\}\)\(B=\{x|x^2+2(a+1)x+a^2-1<0\}\),其中\(x∈R\),如果\(A∩B=B\),求实数\(a\)的取值范围.
 
 

6.设集合\(A=\{x|x^2-ax+a^2-19=0\}\),\(B=\{x|x^2-5x+6=0\}\),\(C=\{x|x^2+2x-8=0\}\)
(1)若\(A∩B=A∪B\),求\(a\)的值
(2)若\(∅⊂A∩B,A∩C=∅\),求\(a\)的值.
 
 

7.已知集合\(A=\{x∣x^2-ax+a^2-19=0\}\)\(B=\{x∣x^2-5x+6=0\}\),是否存在\(a\)使\(A,B\)同时满足下列三个条件:(1)\(A≠B\);(2)\(A∪B=B\);(3)\(∅⫋(A∩B)\).若存在,求出\(a\)的值;若不存在,请说明理由.
 
 

参考答案

  1. 答案 \(B\)
    解析 \(S=\{2,4,6\}\)\(T=\{0,1,2\}\)\(\therefore \dfrac{S}{T}=\{1,2,3,4,6\}\)
    \(\therefore \dfrac{S}{T} \cup T=\{0,1,2,3,4,6\}\)\(∴\)集合\(\dfrac{S}{T} \cup T\)中的元素个数为\(6\)
    故选:\(B\)
  2. 答案 \(A\)
    解析 因为\(A∩B≠∅\),当集合\(A∩B\)中仅有一个元素时,
    集合\((∁_U A)∩(∁_U B)=∁_U (A∪B)\)中有\(3\)个元素,
    \(A∩B\)中有\(6\)个元素时,
    集合\((∁_U A)∩(∁_U B)=∁_U (A∪B)\)中有\(8\)个元素,
    所以得到\(3≤x≤8\)\(x\)为正整数.
    故选:\(A\)
  3. 答案 \(\{0,1,3,4\}\)
    解析 求解一元二次方程\(x^2-(a+3)x+3a=0\)可得\(x_1=a\)\(x_2=3\),且\(B=\{1,4\}\)
    \(a=1\)\(3\)\(4\)时,结合集合的互异性,可知\(A∪B\)中所有元素之和为\(8\)
    否则\(a+1+3+4=8\),解得:\(a=0\)
    综上可得,实数\(a\)的取值范围是\(\{0,1,3,4\}\)
  4. 答案 \(\dfrac{1}{12}\)
    解析 \(m≥0\),且\(m+\dfrac{1}{3} \leq 1\),求出\(m \in\left[0, \dfrac{2}{3}\right]\)
    \(n-\dfrac{3}{4} \geq 0\),且\(n≤1\),求出\(n \in\left[\dfrac{3}{4}, 1\right]\)
    分别把\(m,n\)的两端值代入求出:\(M=\left\{x \mid 0 \leq x \leq \dfrac{1}{3}\right\}\),\(N=\left\{x \mid \dfrac{1}{4} \leq x \leq 1\right\}\)
    \(M=\left\{x \mid \dfrac{2}{3} \leq x \leq 1\right\}\)\(N=\left\{x \mid 0 \leq x \leq \dfrac{3}{4}\right\}\)
    所以\(M \cap N=\left\{x \mid \dfrac{1}{4} \leq x \leq \dfrac{1}{3}\right\}\),或\(\left\{x \mid \dfrac{2}{3} \leq x \leq \dfrac{3}{4}\right\}\)
    所以\(b-a=\dfrac{1}{3}-\dfrac{1}{4}=\dfrac{1}{12}\),或\(\dfrac{3}{4}-\dfrac{2}{3}=\dfrac{1}{12}\)
    综上所述,集合\(M∩N\)的长度的最小值是\(\dfrac{1}{12}\)
  5. 答案 \(a≤-1\)\(a=1\)
    解析 \(A=\{x|-4≤x≤0\}\),又\(A∩B=B\)\(∴B⊆A\)
    \((i)B=∅\)时,\(△=4(a+1)^2-4(a^2-1)≤0\),得\(a≤-1\)
    \((ii)B≠∅\)时,设\(f(x)=x^2+2(a+1)x+a^2-1\)
    \(x^2+2(a+1)x+a^2-1=0\)的两根为\(x_1,x_2\),且\(x_1<x_2\)
    则有\(-4≤x_1<x_2≤0\),即\(\left\{\begin{array}{l} \Delta>0 \\ f(-4) \geq 0 \\ f(0) \geq 0 \\ -4 \leq-(a+1) \leq 0 \end{array}\right.\),解得\(a=1\)
    综上,\(a\)的范围是\(a≤-1\)\(a=1\)
  6. 答案 (1)\(5\) (2)\(-2\)
    解析 (1)集合\(A=\{x|x^2-ax+a^2-19=0\}\),\(B=\{x|x^2-5x+6=0\}=\{2,3\}\)
    \(A∩B=A∪B\),可得\(A=B\)
    \(a=2+3\),且\(a^2-19=2×3\),解得\(a=5\)
    (2)若\(∅⊂A∩B\)\(A∩C=∅\)
    \(C=\{x∣x^2+2x-8=0\}=\{-4,2\}\)\(B=\{2,3\}\)
    可得\(A∩B≠∅\),则\(3∈A\)\(2∉A\)
    可得\(9-3a+a^2-19=0\),解得\(a=5\)\(-2\)
    \(A=\{x|x^2-5x+6=0\}=B\),矛盾,舍去\(a=5\)
    \(A=\{x|x^2+2x-15=0\}=\{-5,3\}\)成立,
    \(a=-2\)
  7. 答案 不存在实数\(a\)使得\(A,B\)满足条件
    解析 假设存在\(a\)使得\(A,B\)满足条件,由题已知得\(B=\{2,3\}\)
    \(∵A∪B=B\),\(∴A⊆B\),即\(A=B\)\(A⫋B\)
    由条件(1)\(A≠B\),可知\(A⫋B\)
    \(∵\emptyset \subsetneq(A \cap B)\)\(∴A≠∅\),即\(A=\{2\}\)\(\{3\}\)
    \(A=\{2\}\)时,代入得\(a^2-2a-15=0\),即\(a=-3\)\(a=5\)
    经检验:\(a=-3\)时,\(A=\{2,-5\}\),与\(A=\{2\}\)矛盾,舍去;
    \(a=5\)时,\(A=\{2,3\}\),与\(A=\{2\}\)矛盾,舍去.
    \(A=\{3\}\)时,代入得\(a^2-3a-10=0\).即\(a=5\)\(a=-2\)
    经检验:\(a=-2\)时,\(A=\{3,-5\}\),与\(A=\{3\}\)矛盾,舍去;
    \(a=5\)时,\(A=\{2,3\}\),与\(A=\{3\}\)矛盾,舍去.
    综上所述,不存在实数\(a\)使得\(A,B\)满足条件.

【C组---拓展题】

1.用\(d(A)\)表示集合\(A\)中的元素个数,若集合\(A=\{0,1\}\),\(B=\{x|(x^2-ax)(x^2-ax+1)=0\}\),且\(|d(A)-d(B)|=1\).设实数\(a\)的所有可能取值构成集合\(M\),则\(d(M)=\) \(\underline{\quad \quad}\).
 

2.定义:给定整数\(i\),如果非空集合\(A\)满足如下\(3\)个条件:
\(A⊆N^*\)
\(A≠\{1\}\)
\(∀x,y∈N^*\),若\(x+y∈A\),则\(xy-i∈A\)
则称集合\(A\)为“减\(i\)集”
(Ⅰ)\(P=\{1,2\}\)是否为“减\(0\)集”?是否为“减\(1\)集”?
(Ⅱ)证明:不存在“减\(2\)集”;
(Ⅲ)是否存在“减\(1\)集”?如果存在,求出所有的“减\(1\)集”;如果不存在,请说明理由.
 
 

参考答案

  1. 答案 \(3\)
    解析 由题意,\(|d(A)-d(B)|=1\)\(d(A)=2\),可得\(d(B)\)的值为\(1\)\(3\)
    \(d(B)=1\),则\(x^2-ax=0\)仅有一根,必为\(0\),此时\(a=0\)
    \(x^2-ax+1=x^2+1=0\)无根,符合题意
    \(d(B)=3\),则\(x^2-ax=0\)有一根,必为\(0\),此时\(a=0\)
    \(x^2-ax+1=x^2+1=0\)无根,不合题意
    \(x^2-ax=0\)有二根,一根是\(0\),另一根是\(a\)
    所以\(x^2-ax+1=0\)必仅有一根,所以\(△=a^2-4=0\),解得\(a=±2\)
    此时\(x^2-ax+1=0\)的解为\(1\)\(-1\),符合题意
    综上实数\(a\)的所有可能取值构成集合\(M=\{0,-2,2\}\),故\(d(M)=3\)
  2. 答案 (1) \(P\)是“减\(0\)集”, 不是“减\(1\)集” (2) 略 (3) \(\{1,3\}\)\(\{1,3,5\}\)\(\{1,3,5,7\}\),…….
    解析 (Ⅰ)\(∵P⊆N^*\)\(P≠\{1\}\)\(1+1=2∈P\)\(1×1-0∈P\)\(∴P\)是“减\(0\)集”
    同理,\(∵P⊆N^*\)\(P≠\{1\}\)\(1+1=2∈P\)\(1×1-1∉P\)\(∴P\)不是“减\(1\)集”.
    (Ⅱ)假设存在\(A\)是“减\(2\)集”,则若\(x+y∈A\)
    那么\(xy-2∈A\),①当\(x+y=xy-2\)时,有\((x-1)(y-1)=3\)
    \(x,y\)一个为\(2\),一个为\(4\),所以集合\(A\)中有元素\(6\)
    但是\(3+3∈A\)\(3×3-2∉A\),与\(A\)是“减\(2\)集”,矛盾;
    ②当\(x+y≠xy-2\)时,则\(x+y=xy-1\)或者\(x+y=xy-m(m>2)\)
    \(x+y=xy-1\)\(m=1\)\(M\)为除\(1\)以外的最小元素,
    \(x=M-1\)\(y=1\)时,\(xy-2=M-3\)小于\(M\)
    如果要符合题意必须\(M=4\),此时取\(x=2,y=2,xy-2=2\)不属于\(A\),故不符合题意.
    \(m>2\)时,\((x-1)(y-1)=m+1\),同样得出矛盾.
    综上可得:不存在\(A\)是“减\(2\)集”.
    (Ⅲ)存在“减\(1\)集”\(A\)\(A≠\{1\}\)
    ①假设\(1∈A\),则\(A\)中除了元素\(1\)以外,必然还含有其它元素.
    假设\(2∈A\)\(1+1∈A\),而\(1×1-1∉A\),因此\(2∉A\)
    假设\(3∈A\)\(1+2∈A\),而\(1×2-1∈A\),因此\(3∈A\)
    因此可以有\(A=\{1,3\}\)
    假设\(4∈A\)\(1+3∈A\),而\(1×3-1∉A\),因此\(4∉A\)
    假设\(5∈A\)\(1+4∈A\)\(1×4-1∈A\)\(2+3=5\)\(2×3-1∈A\),因此\(5∈A\)
    因此可以有\(A=\{1,3,5\}\)
    以此类推可得:\(A=\{1,3,5,……,2n-1,……\}\)\((n∈N^*)\)
    以及A的满足以下条件的非空子集:\(\{1,3\}\)\(\{1,3,5\}\)\(\{1,3,5,7\}\),…….
posted @ 2022-08-30 11:34  贵哥讲数学  阅读(349)  评论(0编辑  收藏  举报
//更改网页ico // 实现数学符号与汉字间有间隙 //文章页加大页面,隐藏侧边栏